Is there a counter-example to these number theoretic conjectures?












2












$begingroup$


Question and Summary



I recently made the following heuristic observations:



Let,
$$ xy = p_1^{a_1} p_2^{a_2} dots p_n^{a_n} $$
where $a_igeq1$



Conjecture $1$: then there must exist $x-y=p_{n+1}$ where $x$ and $y$ co-prime to each other.



Conjecture $2$: for all solutions of different $(a_1,dots,a_n)$ of $x-y= p_{n+1}$ then $x+y$ must also be a prime, $x+y = p_k$.



Can anyone provide any counter examples or reference anything relevant?




$1$st Conjecture



Let the product of $x$ and $y$ be:



$$ xy = p_1^{a_1} p_2^{a_2} dots p_n^{a_n} $$



Where $p_i$ is the $i$'th the prime and $a_i$ is the power of the prime such that $a_i geq1$. We choose $x$ and $y$ co-prime to each other I conjecture there must exist $x$ and $y$ such that:



$$ x-y = p_{n+1}$$



For example given the first prime is $2$:



Where then we get the next prime:
$$ 2^2 - 1 = 3 implies xy = 2^2$$
For the next prime:
$$ 2 . 3 -1 = 5 implies xy = 2.3$$
OR
$$ 3^2 - 2^2 = 5 implies xy = 3^2 2^2$$
OR
$$ 2^5 - 3^3 = 5 implies xy = 3^3 2^5 $$
And so on ... I believe any prime can be constructed by this.




$2$nd Conjecture



However, I also observed if $x-y=p_{n+1}$ then $x+y= p_k$ usually I would think this to only be true if $p_k < p_{n+1}^2$ but I think this might be true even beyond that (?).



For example:



$$ 2^5 + 3^3 = 59 > 25 $$



Implications



If both conjectures are true one can easily say Goldbach's conjecture and twin prime conjecture must be true!










share|cite|improve this question











$endgroup$












  • $begingroup$
    Reason for down vote? Was there a simple counter-example I missed?
    $endgroup$
    – More Anonymous
    Jan 20 at 19:10










  • $begingroup$
    Why should $x-y$ be coprime to $x$ and $y$? I see no assertion that $x$ and $y$ are coprime.
    $endgroup$
    – Matt Samuel
    Jan 20 at 19:11






  • 1




    $begingroup$
    Ah sorry what I meant to say was we choose $x$ and $y$ coprime to each other... and thus $x-y$ is co-prime both to $x$ and $y$ ... I'll edit that
    $endgroup$
    – More Anonymous
    Jan 20 at 19:13








  • 1




    $begingroup$
    The first counterexample I can see for conjecture 2 is when $p = 7$, $x = 2cdot 5^3=250$, and $y=3^5=243$. $xy=493=17cdot 29$. I would bet conjecture 1 is true if I could, but the proof will not be too easy if there is one.
    $endgroup$
    – Hw Chu
    Jan 20 at 20:04






  • 2




    $begingroup$
    I think your Conjecture 1 is equivalent to an idea I had long ago. I asked one of our colloquium speakers about it, and he was unfamiliar with it, but later IIRC I posted it to sci.math (or similar newsgroup) and got a solution that was surprisingly strong. Again if I remember correctly there are primes $p$ that cannot be represented as the difference of two products of powers of distinct primes smaller than $p$. I'll see if Google can help me find that newsgroup thread.
    $endgroup$
    – hardmath
    Jan 20 at 20:20
















2












$begingroup$


Question and Summary



I recently made the following heuristic observations:



Let,
$$ xy = p_1^{a_1} p_2^{a_2} dots p_n^{a_n} $$
where $a_igeq1$



Conjecture $1$: then there must exist $x-y=p_{n+1}$ where $x$ and $y$ co-prime to each other.



Conjecture $2$: for all solutions of different $(a_1,dots,a_n)$ of $x-y= p_{n+1}$ then $x+y$ must also be a prime, $x+y = p_k$.



Can anyone provide any counter examples or reference anything relevant?




$1$st Conjecture



Let the product of $x$ and $y$ be:



$$ xy = p_1^{a_1} p_2^{a_2} dots p_n^{a_n} $$



Where $p_i$ is the $i$'th the prime and $a_i$ is the power of the prime such that $a_i geq1$. We choose $x$ and $y$ co-prime to each other I conjecture there must exist $x$ and $y$ such that:



$$ x-y = p_{n+1}$$



For example given the first prime is $2$:



Where then we get the next prime:
$$ 2^2 - 1 = 3 implies xy = 2^2$$
For the next prime:
$$ 2 . 3 -1 = 5 implies xy = 2.3$$
OR
$$ 3^2 - 2^2 = 5 implies xy = 3^2 2^2$$
OR
$$ 2^5 - 3^3 = 5 implies xy = 3^3 2^5 $$
And so on ... I believe any prime can be constructed by this.




$2$nd Conjecture



However, I also observed if $x-y=p_{n+1}$ then $x+y= p_k$ usually I would think this to only be true if $p_k < p_{n+1}^2$ but I think this might be true even beyond that (?).



For example:



$$ 2^5 + 3^3 = 59 > 25 $$



Implications



If both conjectures are true one can easily say Goldbach's conjecture and twin prime conjecture must be true!










share|cite|improve this question











$endgroup$












  • $begingroup$
    Reason for down vote? Was there a simple counter-example I missed?
    $endgroup$
    – More Anonymous
    Jan 20 at 19:10










  • $begingroup$
    Why should $x-y$ be coprime to $x$ and $y$? I see no assertion that $x$ and $y$ are coprime.
    $endgroup$
    – Matt Samuel
    Jan 20 at 19:11






  • 1




    $begingroup$
    Ah sorry what I meant to say was we choose $x$ and $y$ coprime to each other... and thus $x-y$ is co-prime both to $x$ and $y$ ... I'll edit that
    $endgroup$
    – More Anonymous
    Jan 20 at 19:13








  • 1




    $begingroup$
    The first counterexample I can see for conjecture 2 is when $p = 7$, $x = 2cdot 5^3=250$, and $y=3^5=243$. $xy=493=17cdot 29$. I would bet conjecture 1 is true if I could, but the proof will not be too easy if there is one.
    $endgroup$
    – Hw Chu
    Jan 20 at 20:04






  • 2




    $begingroup$
    I think your Conjecture 1 is equivalent to an idea I had long ago. I asked one of our colloquium speakers about it, and he was unfamiliar with it, but later IIRC I posted it to sci.math (or similar newsgroup) and got a solution that was surprisingly strong. Again if I remember correctly there are primes $p$ that cannot be represented as the difference of two products of powers of distinct primes smaller than $p$. I'll see if Google can help me find that newsgroup thread.
    $endgroup$
    – hardmath
    Jan 20 at 20:20














2












2








2


2



$begingroup$


Question and Summary



I recently made the following heuristic observations:



Let,
$$ xy = p_1^{a_1} p_2^{a_2} dots p_n^{a_n} $$
where $a_igeq1$



Conjecture $1$: then there must exist $x-y=p_{n+1}$ where $x$ and $y$ co-prime to each other.



Conjecture $2$: for all solutions of different $(a_1,dots,a_n)$ of $x-y= p_{n+1}$ then $x+y$ must also be a prime, $x+y = p_k$.



Can anyone provide any counter examples or reference anything relevant?




$1$st Conjecture



Let the product of $x$ and $y$ be:



$$ xy = p_1^{a_1} p_2^{a_2} dots p_n^{a_n} $$



Where $p_i$ is the $i$'th the prime and $a_i$ is the power of the prime such that $a_i geq1$. We choose $x$ and $y$ co-prime to each other I conjecture there must exist $x$ and $y$ such that:



$$ x-y = p_{n+1}$$



For example given the first prime is $2$:



Where then we get the next prime:
$$ 2^2 - 1 = 3 implies xy = 2^2$$
For the next prime:
$$ 2 . 3 -1 = 5 implies xy = 2.3$$
OR
$$ 3^2 - 2^2 = 5 implies xy = 3^2 2^2$$
OR
$$ 2^5 - 3^3 = 5 implies xy = 3^3 2^5 $$
And so on ... I believe any prime can be constructed by this.




$2$nd Conjecture



However, I also observed if $x-y=p_{n+1}$ then $x+y= p_k$ usually I would think this to only be true if $p_k < p_{n+1}^2$ but I think this might be true even beyond that (?).



For example:



$$ 2^5 + 3^3 = 59 > 25 $$



Implications



If both conjectures are true one can easily say Goldbach's conjecture and twin prime conjecture must be true!










share|cite|improve this question











$endgroup$




Question and Summary



I recently made the following heuristic observations:



Let,
$$ xy = p_1^{a_1} p_2^{a_2} dots p_n^{a_n} $$
where $a_igeq1$



Conjecture $1$: then there must exist $x-y=p_{n+1}$ where $x$ and $y$ co-prime to each other.



Conjecture $2$: for all solutions of different $(a_1,dots,a_n)$ of $x-y= p_{n+1}$ then $x+y$ must also be a prime, $x+y = p_k$.



Can anyone provide any counter examples or reference anything relevant?




$1$st Conjecture



Let the product of $x$ and $y$ be:



$$ xy = p_1^{a_1} p_2^{a_2} dots p_n^{a_n} $$



Where $p_i$ is the $i$'th the prime and $a_i$ is the power of the prime such that $a_i geq1$. We choose $x$ and $y$ co-prime to each other I conjecture there must exist $x$ and $y$ such that:



$$ x-y = p_{n+1}$$



For example given the first prime is $2$:



Where then we get the next prime:
$$ 2^2 - 1 = 3 implies xy = 2^2$$
For the next prime:
$$ 2 . 3 -1 = 5 implies xy = 2.3$$
OR
$$ 3^2 - 2^2 = 5 implies xy = 3^2 2^2$$
OR
$$ 2^5 - 3^3 = 5 implies xy = 3^3 2^5 $$
And so on ... I believe any prime can be constructed by this.




$2$nd Conjecture



However, I also observed if $x-y=p_{n+1}$ then $x+y= p_k$ usually I would think this to only be true if $p_k < p_{n+1}^2$ but I think this might be true even beyond that (?).



For example:



$$ 2^5 + 3^3 = 59 > 25 $$



Implications



If both conjectures are true one can easily say Goldbach's conjecture and twin prime conjecture must be true!







number-theory reference-request conjectures prime-twins goldbachs-conjecture






share|cite|improve this question















share|cite|improve this question













share|cite|improve this question




share|cite|improve this question








edited Jan 20 at 19:16







More Anonymous

















asked Jan 20 at 19:05









More AnonymousMore Anonymous

36019




36019












  • $begingroup$
    Reason for down vote? Was there a simple counter-example I missed?
    $endgroup$
    – More Anonymous
    Jan 20 at 19:10










  • $begingroup$
    Why should $x-y$ be coprime to $x$ and $y$? I see no assertion that $x$ and $y$ are coprime.
    $endgroup$
    – Matt Samuel
    Jan 20 at 19:11






  • 1




    $begingroup$
    Ah sorry what I meant to say was we choose $x$ and $y$ coprime to each other... and thus $x-y$ is co-prime both to $x$ and $y$ ... I'll edit that
    $endgroup$
    – More Anonymous
    Jan 20 at 19:13








  • 1




    $begingroup$
    The first counterexample I can see for conjecture 2 is when $p = 7$, $x = 2cdot 5^3=250$, and $y=3^5=243$. $xy=493=17cdot 29$. I would bet conjecture 1 is true if I could, but the proof will not be too easy if there is one.
    $endgroup$
    – Hw Chu
    Jan 20 at 20:04






  • 2




    $begingroup$
    I think your Conjecture 1 is equivalent to an idea I had long ago. I asked one of our colloquium speakers about it, and he was unfamiliar with it, but later IIRC I posted it to sci.math (or similar newsgroup) and got a solution that was surprisingly strong. Again if I remember correctly there are primes $p$ that cannot be represented as the difference of two products of powers of distinct primes smaller than $p$. I'll see if Google can help me find that newsgroup thread.
    $endgroup$
    – hardmath
    Jan 20 at 20:20


















  • $begingroup$
    Reason for down vote? Was there a simple counter-example I missed?
    $endgroup$
    – More Anonymous
    Jan 20 at 19:10










  • $begingroup$
    Why should $x-y$ be coprime to $x$ and $y$? I see no assertion that $x$ and $y$ are coprime.
    $endgroup$
    – Matt Samuel
    Jan 20 at 19:11






  • 1




    $begingroup$
    Ah sorry what I meant to say was we choose $x$ and $y$ coprime to each other... and thus $x-y$ is co-prime both to $x$ and $y$ ... I'll edit that
    $endgroup$
    – More Anonymous
    Jan 20 at 19:13








  • 1




    $begingroup$
    The first counterexample I can see for conjecture 2 is when $p = 7$, $x = 2cdot 5^3=250$, and $y=3^5=243$. $xy=493=17cdot 29$. I would bet conjecture 1 is true if I could, but the proof will not be too easy if there is one.
    $endgroup$
    – Hw Chu
    Jan 20 at 20:04






  • 2




    $begingroup$
    I think your Conjecture 1 is equivalent to an idea I had long ago. I asked one of our colloquium speakers about it, and he was unfamiliar with it, but later IIRC I posted it to sci.math (or similar newsgroup) and got a solution that was surprisingly strong. Again if I remember correctly there are primes $p$ that cannot be represented as the difference of two products of powers of distinct primes smaller than $p$. I'll see if Google can help me find that newsgroup thread.
    $endgroup$
    – hardmath
    Jan 20 at 20:20
















$begingroup$
Reason for down vote? Was there a simple counter-example I missed?
$endgroup$
– More Anonymous
Jan 20 at 19:10




$begingroup$
Reason for down vote? Was there a simple counter-example I missed?
$endgroup$
– More Anonymous
Jan 20 at 19:10












$begingroup$
Why should $x-y$ be coprime to $x$ and $y$? I see no assertion that $x$ and $y$ are coprime.
$endgroup$
– Matt Samuel
Jan 20 at 19:11




$begingroup$
Why should $x-y$ be coprime to $x$ and $y$? I see no assertion that $x$ and $y$ are coprime.
$endgroup$
– Matt Samuel
Jan 20 at 19:11




1




1




$begingroup$
Ah sorry what I meant to say was we choose $x$ and $y$ coprime to each other... and thus $x-y$ is co-prime both to $x$ and $y$ ... I'll edit that
$endgroup$
– More Anonymous
Jan 20 at 19:13






$begingroup$
Ah sorry what I meant to say was we choose $x$ and $y$ coprime to each other... and thus $x-y$ is co-prime both to $x$ and $y$ ... I'll edit that
$endgroup$
– More Anonymous
Jan 20 at 19:13






1




1




$begingroup$
The first counterexample I can see for conjecture 2 is when $p = 7$, $x = 2cdot 5^3=250$, and $y=3^5=243$. $xy=493=17cdot 29$. I would bet conjecture 1 is true if I could, but the proof will not be too easy if there is one.
$endgroup$
– Hw Chu
Jan 20 at 20:04




$begingroup$
The first counterexample I can see for conjecture 2 is when $p = 7$, $x = 2cdot 5^3=250$, and $y=3^5=243$. $xy=493=17cdot 29$. I would bet conjecture 1 is true if I could, but the proof will not be too easy if there is one.
$endgroup$
– Hw Chu
Jan 20 at 20:04




2




2




$begingroup$
I think your Conjecture 1 is equivalent to an idea I had long ago. I asked one of our colloquium speakers about it, and he was unfamiliar with it, but later IIRC I posted it to sci.math (or similar newsgroup) and got a solution that was surprisingly strong. Again if I remember correctly there are primes $p$ that cannot be represented as the difference of two products of powers of distinct primes smaller than $p$. I'll see if Google can help me find that newsgroup thread.
$endgroup$
– hardmath
Jan 20 at 20:20




$begingroup$
I think your Conjecture 1 is equivalent to an idea I had long ago. I asked one of our colloquium speakers about it, and he was unfamiliar with it, but later IIRC I posted it to sci.math (or similar newsgroup) and got a solution that was surprisingly strong. Again if I remember correctly there are primes $p$ that cannot be represented as the difference of two products of powers of distinct primes smaller than $p$. I'll see if Google can help me find that newsgroup thread.
$endgroup$
– hardmath
Jan 20 at 20:20










3 Answers
3






active

oldest

votes


















4












$begingroup$

I strongly suspect that you have tried only very small $n$. The minimal product $xy$ grows very rapidly. For example, for $n=10$ the minimum product $xy$ is $6469693230$, the product of the primes up to $29$. That would mean $x$ and $y$ are about $80434$. The first case where there are no $x,y$ with all exponents $1$ is $n=7$, where we would want $y(y+19)=510510$. It does not have a solution in the integers. There might be a solution with larger exponents, but I am skeptical. I was surprised to find $n=6$ works with $x=182, y=165, xy=30030$



Your second conjecture will be true when the exponents are small enough. You know that $x+y$ has no prime factors $p_{n+1}$ or smaller, so you are guaranteed that it is prime when $x+y lt p_{n+1}^2$. However, as you get more primes, $x$ and $y$ will increase because their product is at least the primorial. Say we let $n=10$, so the primes are $2,3,5,7,11,13,17,19,23,29$. We have $xy$ is at least $6469693230$, so if we can use all first powers we have $x,y$ are about $80434$ and $x+y$ will be about $160868$, so there are lots of primes that could be factors.



I don't see how these two would solve Goldbach's conjecture or the twin primes problem, but if they would I would be even more surprised if they were true.






share|cite|improve this answer











$endgroup$





















    1












    $begingroup$

    This post suggests an approach to the 1st Conjecture, too long for a comment. It is not intended to be a full answer.



    OP requires $alpha_i ge 1$. Let $s:=prod{q_j}; t:= prod {r_k}; (q_j,r_k)in mathbb P; q_j ne r_k; st=p_n#$



    Plainly $gcd(s,t)=1$, so there exist $a,b$ such that $(as-bt)=|1|$



    Accordingly, there will be $c,d$ such that $(cs-dt)=|p_{n+1}|$. We can let $x,y$ correspond to appropriate terms of $(cs-dt)$ or $(dt-cs)$ so as to conform to the nomenclature of the 1st Conjecture and result in a positive value for $p_{n+1}$.



    The question devolves to whether $c$ can be found which contains only prime factors among $q_i$ and simultaneously $d$ can be found which contains only prime factors among $r_j$



    If I make progress on this, I will update this post; I invite others to present solutions if they are aware of any.






    share|cite|improve this answer









    $endgroup$





















      0












      $begingroup$

      I am providing this as an answer rather than as a comment as it's too long for that. Note there's a fairly similar question from Sep. $10, 2015$ on MathOverflow of Constructing prime numbers. The differences are that this other question supports $x pm y = p_{n+1}$, plus for $xy = p_1^{a_1}p_2^{a_2}ldots p_n^{a_n}$, it requires the $a_i = 1$ rather than allowing $a_i ge 1$.



      However, one of the answers, the $2$nd one down as I'm writing this, by Asterios Gkantzounis, says




      maybe you could take the primes on the products in any powers you want , it is a thought that i had before some years and it is also a natural question that one can make reading Euclid's proof since still the difference of the products is not divisible by any of these primes do you take in this way always the next prime?




      I believe this is basically what you are asking. A comment by François Brunault says




      If you allow arbitrary exponents (and if I understand well your suggestion) then it is indeed always possible to get the next prime, just because of Fermat's little theorem. More complicated explanation : $left(text{Z}/ptext{Z}right)^{*}$ is generated by the classes of the primes $< p$.




      As my knowledge of number theory is quite limited, I can't really comment on how potentially valid his comment is. However, his profile indicates he's an experienced number theorist, so I don't have any reason to doubt his statement, although hardmath's comment to the question indicates it might not be true (of course, if I'm understanding everything correctly).



      Note I am leaving a comment to Asterios that you're also asking a very similar question here.






      share|cite|improve this answer











      $endgroup$













        Your Answer





        StackExchange.ifUsing("editor", function () {
        return StackExchange.using("mathjaxEditing", function () {
        StackExchange.MarkdownEditor.creationCallbacks.add(function (editor, postfix) {
        StackExchange.mathjaxEditing.prepareWmdForMathJax(editor, postfix, [["$", "$"], ["\\(","\\)"]]);
        });
        });
        }, "mathjax-editing");

        StackExchange.ready(function() {
        var channelOptions = {
        tags: "".split(" "),
        id: "69"
        };
        initTagRenderer("".split(" "), "".split(" "), channelOptions);

        StackExchange.using("externalEditor", function() {
        // Have to fire editor after snippets, if snippets enabled
        if (StackExchange.settings.snippets.snippetsEnabled) {
        StackExchange.using("snippets", function() {
        createEditor();
        });
        }
        else {
        createEditor();
        }
        });

        function createEditor() {
        StackExchange.prepareEditor({
        heartbeatType: 'answer',
        autoActivateHeartbeat: false,
        convertImagesToLinks: true,
        noModals: true,
        showLowRepImageUploadWarning: true,
        reputationToPostImages: 10,
        bindNavPrevention: true,
        postfix: "",
        imageUploader: {
        brandingHtml: "Powered by u003ca class="icon-imgur-white" href="https://imgur.com/"u003eu003c/au003e",
        contentPolicyHtml: "User contributions licensed under u003ca href="https://creativecommons.org/licenses/by-sa/3.0/"u003ecc by-sa 3.0 with attribution requiredu003c/au003e u003ca href="https://stackoverflow.com/legal/content-policy"u003e(content policy)u003c/au003e",
        allowUrls: true
        },
        noCode: true, onDemand: true,
        discardSelector: ".discard-answer"
        ,immediatelyShowMarkdownHelp:true
        });


        }
        });














        draft saved

        draft discarded


















        StackExchange.ready(
        function () {
        StackExchange.openid.initPostLogin('.new-post-login', 'https%3a%2f%2fmath.stackexchange.com%2fquestions%2f3081009%2fis-there-a-counter-example-to-these-number-theoretic-conjectures%23new-answer', 'question_page');
        }
        );

        Post as a guest















        Required, but never shown

























        3 Answers
        3






        active

        oldest

        votes








        3 Answers
        3






        active

        oldest

        votes









        active

        oldest

        votes






        active

        oldest

        votes









        4












        $begingroup$

        I strongly suspect that you have tried only very small $n$. The minimal product $xy$ grows very rapidly. For example, for $n=10$ the minimum product $xy$ is $6469693230$, the product of the primes up to $29$. That would mean $x$ and $y$ are about $80434$. The first case where there are no $x,y$ with all exponents $1$ is $n=7$, where we would want $y(y+19)=510510$. It does not have a solution in the integers. There might be a solution with larger exponents, but I am skeptical. I was surprised to find $n=6$ works with $x=182, y=165, xy=30030$



        Your second conjecture will be true when the exponents are small enough. You know that $x+y$ has no prime factors $p_{n+1}$ or smaller, so you are guaranteed that it is prime when $x+y lt p_{n+1}^2$. However, as you get more primes, $x$ and $y$ will increase because their product is at least the primorial. Say we let $n=10$, so the primes are $2,3,5,7,11,13,17,19,23,29$. We have $xy$ is at least $6469693230$, so if we can use all first powers we have $x,y$ are about $80434$ and $x+y$ will be about $160868$, so there are lots of primes that could be factors.



        I don't see how these two would solve Goldbach's conjecture or the twin primes problem, but if they would I would be even more surprised if they were true.






        share|cite|improve this answer











        $endgroup$


















          4












          $begingroup$

          I strongly suspect that you have tried only very small $n$. The minimal product $xy$ grows very rapidly. For example, for $n=10$ the minimum product $xy$ is $6469693230$, the product of the primes up to $29$. That would mean $x$ and $y$ are about $80434$. The first case where there are no $x,y$ with all exponents $1$ is $n=7$, where we would want $y(y+19)=510510$. It does not have a solution in the integers. There might be a solution with larger exponents, but I am skeptical. I was surprised to find $n=6$ works with $x=182, y=165, xy=30030$



          Your second conjecture will be true when the exponents are small enough. You know that $x+y$ has no prime factors $p_{n+1}$ or smaller, so you are guaranteed that it is prime when $x+y lt p_{n+1}^2$. However, as you get more primes, $x$ and $y$ will increase because their product is at least the primorial. Say we let $n=10$, so the primes are $2,3,5,7,11,13,17,19,23,29$. We have $xy$ is at least $6469693230$, so if we can use all first powers we have $x,y$ are about $80434$ and $x+y$ will be about $160868$, so there are lots of primes that could be factors.



          I don't see how these two would solve Goldbach's conjecture or the twin primes problem, but if they would I would be even more surprised if they were true.






          share|cite|improve this answer











          $endgroup$
















            4












            4








            4





            $begingroup$

            I strongly suspect that you have tried only very small $n$. The minimal product $xy$ grows very rapidly. For example, for $n=10$ the minimum product $xy$ is $6469693230$, the product of the primes up to $29$. That would mean $x$ and $y$ are about $80434$. The first case where there are no $x,y$ with all exponents $1$ is $n=7$, where we would want $y(y+19)=510510$. It does not have a solution in the integers. There might be a solution with larger exponents, but I am skeptical. I was surprised to find $n=6$ works with $x=182, y=165, xy=30030$



            Your second conjecture will be true when the exponents are small enough. You know that $x+y$ has no prime factors $p_{n+1}$ or smaller, so you are guaranteed that it is prime when $x+y lt p_{n+1}^2$. However, as you get more primes, $x$ and $y$ will increase because their product is at least the primorial. Say we let $n=10$, so the primes are $2,3,5,7,11,13,17,19,23,29$. We have $xy$ is at least $6469693230$, so if we can use all first powers we have $x,y$ are about $80434$ and $x+y$ will be about $160868$, so there are lots of primes that could be factors.



            I don't see how these two would solve Goldbach's conjecture or the twin primes problem, but if they would I would be even more surprised if they were true.






            share|cite|improve this answer











            $endgroup$



            I strongly suspect that you have tried only very small $n$. The minimal product $xy$ grows very rapidly. For example, for $n=10$ the minimum product $xy$ is $6469693230$, the product of the primes up to $29$. That would mean $x$ and $y$ are about $80434$. The first case where there are no $x,y$ with all exponents $1$ is $n=7$, where we would want $y(y+19)=510510$. It does not have a solution in the integers. There might be a solution with larger exponents, but I am skeptical. I was surprised to find $n=6$ works with $x=182, y=165, xy=30030$



            Your second conjecture will be true when the exponents are small enough. You know that $x+y$ has no prime factors $p_{n+1}$ or smaller, so you are guaranteed that it is prime when $x+y lt p_{n+1}^2$. However, as you get more primes, $x$ and $y$ will increase because their product is at least the primorial. Say we let $n=10$, so the primes are $2,3,5,7,11,13,17,19,23,29$. We have $xy$ is at least $6469693230$, so if we can use all first powers we have $x,y$ are about $80434$ and $x+y$ will be about $160868$, so there are lots of primes that could be factors.



            I don't see how these two would solve Goldbach's conjecture or the twin primes problem, but if they would I would be even more surprised if they were true.







            share|cite|improve this answer














            share|cite|improve this answer



            share|cite|improve this answer








            edited Jan 20 at 20:13

























            answered Jan 20 at 19:46









            Ross MillikanRoss Millikan

            297k23198371




            297k23198371























                1












                $begingroup$

                This post suggests an approach to the 1st Conjecture, too long for a comment. It is not intended to be a full answer.



                OP requires $alpha_i ge 1$. Let $s:=prod{q_j}; t:= prod {r_k}; (q_j,r_k)in mathbb P; q_j ne r_k; st=p_n#$



                Plainly $gcd(s,t)=1$, so there exist $a,b$ such that $(as-bt)=|1|$



                Accordingly, there will be $c,d$ such that $(cs-dt)=|p_{n+1}|$. We can let $x,y$ correspond to appropriate terms of $(cs-dt)$ or $(dt-cs)$ so as to conform to the nomenclature of the 1st Conjecture and result in a positive value for $p_{n+1}$.



                The question devolves to whether $c$ can be found which contains only prime factors among $q_i$ and simultaneously $d$ can be found which contains only prime factors among $r_j$



                If I make progress on this, I will update this post; I invite others to present solutions if they are aware of any.






                share|cite|improve this answer









                $endgroup$


















                  1












                  $begingroup$

                  This post suggests an approach to the 1st Conjecture, too long for a comment. It is not intended to be a full answer.



                  OP requires $alpha_i ge 1$. Let $s:=prod{q_j}; t:= prod {r_k}; (q_j,r_k)in mathbb P; q_j ne r_k; st=p_n#$



                  Plainly $gcd(s,t)=1$, so there exist $a,b$ such that $(as-bt)=|1|$



                  Accordingly, there will be $c,d$ such that $(cs-dt)=|p_{n+1}|$. We can let $x,y$ correspond to appropriate terms of $(cs-dt)$ or $(dt-cs)$ so as to conform to the nomenclature of the 1st Conjecture and result in a positive value for $p_{n+1}$.



                  The question devolves to whether $c$ can be found which contains only prime factors among $q_i$ and simultaneously $d$ can be found which contains only prime factors among $r_j$



                  If I make progress on this, I will update this post; I invite others to present solutions if they are aware of any.






                  share|cite|improve this answer









                  $endgroup$
















                    1












                    1








                    1





                    $begingroup$

                    This post suggests an approach to the 1st Conjecture, too long for a comment. It is not intended to be a full answer.



                    OP requires $alpha_i ge 1$. Let $s:=prod{q_j}; t:= prod {r_k}; (q_j,r_k)in mathbb P; q_j ne r_k; st=p_n#$



                    Plainly $gcd(s,t)=1$, so there exist $a,b$ such that $(as-bt)=|1|$



                    Accordingly, there will be $c,d$ such that $(cs-dt)=|p_{n+1}|$. We can let $x,y$ correspond to appropriate terms of $(cs-dt)$ or $(dt-cs)$ so as to conform to the nomenclature of the 1st Conjecture and result in a positive value for $p_{n+1}$.



                    The question devolves to whether $c$ can be found which contains only prime factors among $q_i$ and simultaneously $d$ can be found which contains only prime factors among $r_j$



                    If I make progress on this, I will update this post; I invite others to present solutions if they are aware of any.






                    share|cite|improve this answer









                    $endgroup$



                    This post suggests an approach to the 1st Conjecture, too long for a comment. It is not intended to be a full answer.



                    OP requires $alpha_i ge 1$. Let $s:=prod{q_j}; t:= prod {r_k}; (q_j,r_k)in mathbb P; q_j ne r_k; st=p_n#$



                    Plainly $gcd(s,t)=1$, so there exist $a,b$ such that $(as-bt)=|1|$



                    Accordingly, there will be $c,d$ such that $(cs-dt)=|p_{n+1}|$. We can let $x,y$ correspond to appropriate terms of $(cs-dt)$ or $(dt-cs)$ so as to conform to the nomenclature of the 1st Conjecture and result in a positive value for $p_{n+1}$.



                    The question devolves to whether $c$ can be found which contains only prime factors among $q_i$ and simultaneously $d$ can be found which contains only prime factors among $r_j$



                    If I make progress on this, I will update this post; I invite others to present solutions if they are aware of any.







                    share|cite|improve this answer












                    share|cite|improve this answer



                    share|cite|improve this answer










                    answered Jan 21 at 20:52









                    Keith BackmanKeith Backman

                    1,3681812




                    1,3681812























                        0












                        $begingroup$

                        I am providing this as an answer rather than as a comment as it's too long for that. Note there's a fairly similar question from Sep. $10, 2015$ on MathOverflow of Constructing prime numbers. The differences are that this other question supports $x pm y = p_{n+1}$, plus for $xy = p_1^{a_1}p_2^{a_2}ldots p_n^{a_n}$, it requires the $a_i = 1$ rather than allowing $a_i ge 1$.



                        However, one of the answers, the $2$nd one down as I'm writing this, by Asterios Gkantzounis, says




                        maybe you could take the primes on the products in any powers you want , it is a thought that i had before some years and it is also a natural question that one can make reading Euclid's proof since still the difference of the products is not divisible by any of these primes do you take in this way always the next prime?




                        I believe this is basically what you are asking. A comment by François Brunault says




                        If you allow arbitrary exponents (and if I understand well your suggestion) then it is indeed always possible to get the next prime, just because of Fermat's little theorem. More complicated explanation : $left(text{Z}/ptext{Z}right)^{*}$ is generated by the classes of the primes $< p$.




                        As my knowledge of number theory is quite limited, I can't really comment on how potentially valid his comment is. However, his profile indicates he's an experienced number theorist, so I don't have any reason to doubt his statement, although hardmath's comment to the question indicates it might not be true (of course, if I'm understanding everything correctly).



                        Note I am leaving a comment to Asterios that you're also asking a very similar question here.






                        share|cite|improve this answer











                        $endgroup$


















                          0












                          $begingroup$

                          I am providing this as an answer rather than as a comment as it's too long for that. Note there's a fairly similar question from Sep. $10, 2015$ on MathOverflow of Constructing prime numbers. The differences are that this other question supports $x pm y = p_{n+1}$, plus for $xy = p_1^{a_1}p_2^{a_2}ldots p_n^{a_n}$, it requires the $a_i = 1$ rather than allowing $a_i ge 1$.



                          However, one of the answers, the $2$nd one down as I'm writing this, by Asterios Gkantzounis, says




                          maybe you could take the primes on the products in any powers you want , it is a thought that i had before some years and it is also a natural question that one can make reading Euclid's proof since still the difference of the products is not divisible by any of these primes do you take in this way always the next prime?




                          I believe this is basically what you are asking. A comment by François Brunault says




                          If you allow arbitrary exponents (and if I understand well your suggestion) then it is indeed always possible to get the next prime, just because of Fermat's little theorem. More complicated explanation : $left(text{Z}/ptext{Z}right)^{*}$ is generated by the classes of the primes $< p$.




                          As my knowledge of number theory is quite limited, I can't really comment on how potentially valid his comment is. However, his profile indicates he's an experienced number theorist, so I don't have any reason to doubt his statement, although hardmath's comment to the question indicates it might not be true (of course, if I'm understanding everything correctly).



                          Note I am leaving a comment to Asterios that you're also asking a very similar question here.






                          share|cite|improve this answer











                          $endgroup$
















                            0












                            0








                            0





                            $begingroup$

                            I am providing this as an answer rather than as a comment as it's too long for that. Note there's a fairly similar question from Sep. $10, 2015$ on MathOverflow of Constructing prime numbers. The differences are that this other question supports $x pm y = p_{n+1}$, plus for $xy = p_1^{a_1}p_2^{a_2}ldots p_n^{a_n}$, it requires the $a_i = 1$ rather than allowing $a_i ge 1$.



                            However, one of the answers, the $2$nd one down as I'm writing this, by Asterios Gkantzounis, says




                            maybe you could take the primes on the products in any powers you want , it is a thought that i had before some years and it is also a natural question that one can make reading Euclid's proof since still the difference of the products is not divisible by any of these primes do you take in this way always the next prime?




                            I believe this is basically what you are asking. A comment by François Brunault says




                            If you allow arbitrary exponents (and if I understand well your suggestion) then it is indeed always possible to get the next prime, just because of Fermat's little theorem. More complicated explanation : $left(text{Z}/ptext{Z}right)^{*}$ is generated by the classes of the primes $< p$.




                            As my knowledge of number theory is quite limited, I can't really comment on how potentially valid his comment is. However, his profile indicates he's an experienced number theorist, so I don't have any reason to doubt his statement, although hardmath's comment to the question indicates it might not be true (of course, if I'm understanding everything correctly).



                            Note I am leaving a comment to Asterios that you're also asking a very similar question here.






                            share|cite|improve this answer











                            $endgroup$



                            I am providing this as an answer rather than as a comment as it's too long for that. Note there's a fairly similar question from Sep. $10, 2015$ on MathOverflow of Constructing prime numbers. The differences are that this other question supports $x pm y = p_{n+1}$, plus for $xy = p_1^{a_1}p_2^{a_2}ldots p_n^{a_n}$, it requires the $a_i = 1$ rather than allowing $a_i ge 1$.



                            However, one of the answers, the $2$nd one down as I'm writing this, by Asterios Gkantzounis, says




                            maybe you could take the primes on the products in any powers you want , it is a thought that i had before some years and it is also a natural question that one can make reading Euclid's proof since still the difference of the products is not divisible by any of these primes do you take in this way always the next prime?




                            I believe this is basically what you are asking. A comment by François Brunault says




                            If you allow arbitrary exponents (and if I understand well your suggestion) then it is indeed always possible to get the next prime, just because of Fermat's little theorem. More complicated explanation : $left(text{Z}/ptext{Z}right)^{*}$ is generated by the classes of the primes $< p$.




                            As my knowledge of number theory is quite limited, I can't really comment on how potentially valid his comment is. However, his profile indicates he's an experienced number theorist, so I don't have any reason to doubt his statement, although hardmath's comment to the question indicates it might not be true (of course, if I'm understanding everything correctly).



                            Note I am leaving a comment to Asterios that you're also asking a very similar question here.







                            share|cite|improve this answer














                            share|cite|improve this answer



                            share|cite|improve this answer








                            edited Jan 27 at 22:15

























                            answered Jan 27 at 22:08









                            John OmielanJohn Omielan

                            2,950213




                            2,950213






























                                draft saved

                                draft discarded




















































                                Thanks for contributing an answer to Mathematics Stack Exchange!


                                • Please be sure to answer the question. Provide details and share your research!

                                But avoid



                                • Asking for help, clarification, or responding to other answers.

                                • Making statements based on opinion; back them up with references or personal experience.


                                Use MathJax to format equations. MathJax reference.


                                To learn more, see our tips on writing great answers.




                                draft saved


                                draft discarded














                                StackExchange.ready(
                                function () {
                                StackExchange.openid.initPostLogin('.new-post-login', 'https%3a%2f%2fmath.stackexchange.com%2fquestions%2f3081009%2fis-there-a-counter-example-to-these-number-theoretic-conjectures%23new-answer', 'question_page');
                                }
                                );

                                Post as a guest















                                Required, but never shown





















































                                Required, but never shown














                                Required, but never shown












                                Required, but never shown







                                Required, but never shown

































                                Required, but never shown














                                Required, but never shown












                                Required, but never shown







                                Required, but never shown







                                Popular posts from this blog

                                Mario Kart Wii

                                What does “Dominus providebit” mean?

                                Antonio Litta Visconti Arese